Please answer correctly !!!!!!!! Will mark brainliest !!!!!!!!!!!!!

Please Answer Correctly !!!!!!!! Will Mark Brainliest !!!!!!!!!!!!!

Answers

Answer 1

Answer:

B.

Step-by-step explanation:

It is very possible.

Make the value of apple x and the value of orange y.

5x + 4y = 10

5x + 5y = 11

-y = -1

y = 1

x = 1.2

Now just match them up.

Yes; They should charge $1.20 for and apple and $1.00 for an orange.


Related Questions

Mel's Furniture received an invoice dated September 27 for 5 bedroom sets at $3,000 each. The invoice indicated a chain discount of 5/8/3. The seller of the furniture prepaid the freight of $200 (FOB Shipping Point). Terms were 2/10, EOM. Assuming that Mel pays within the discount period, what is the total payment amount?

Answers

Answer: $12,916.70

Step-by-step explanation:

Given the following:

Invoice received :

Bedroom set = 5

Cost per set = $3,000

Chain discount = 5/8/3

Freight cost = $200

If Mel pays within the discount period:

Chain discount given = 5/8/3

Therefore, net equivalent price rate equals:

(1 - 0.05) × (1 - 0.08) × (1 - 0.03) =

0.95 × 0.92 × 0.97 = 0.84778

Net price = total cost × 0.84778

($3000 × 5) × 0.84778

$15000 × 0.84778 = $12,716.7

Net equivalent price + FOB Shipping

$12,716.7 + $200 = $12,916.70

5xthe power of x...................

Answers

Answer:

[tex]5x^{x}[/tex]

If 32 % of a is 1.12 then what is the value of a?

Answers

Answer:

4.5 is the value of A

Step-by-step explanation:

do it doing part/whole and cross crossing :)

32/100 because it’s a percentage and 1.12/x. 1.12 times 100 to get 112, and then 32 times x = 32x. your equation right now is 32x = 112, bur we want to get rid of that x, so we are going to divide 112/32 to get the whole of 3.5 for the value of A.

An exterior angle of an isosceles triangle has a Measure 160°. Find two possible sets of measures for the angles of the triangle. If the exterior angle of the bases is 160° then the measure of the angle of each base is

Answers

Answer:

the angles would either be 20, 20, 140 or 20, 80, 80

Step-by-step explanation:

Answer:

20, 20, 140 or 20, 80, 80

Step-by-step explanation:

A carnival ride is in the shape of a wheel with a radius of 30 feet. The wheel has 30 cars attached to the center of the wheel. What is the central angle, arc length, and area of a sector between any two cars? Round answers to the nearest hundredth if applicable. You must show all work and calculations to receive credit.

Answers

Answer:

[tex]12^{\circ}[/tex]  [tex]1\ feet\ and[/tex] [tex]94.28\ ft^2[/tex]

Step-by-step explanation:

Data provided in the question

The radius of a wheel = 30 feet

Number of cars attached = 30 cars

Based on the above information

The computation is shown below:

The center angle is

[tex]= \frac{360}{30}[/tex]

[tex]= 12^{\circ}[/tex]

The arc length is

[tex]= \frac{12}{360}(30)[/tex]

= 1 feet

And, the area of a sector between two cars is

[tex]= \frac{12}{360}(\pi (30)^2)[/tex]

= [tex]94.28\ ft^2[/tex]

We simply do the above calculations so that each item could be find out

b) Write 1.27 x 10-2 as an
as an ordinary number.​

Answers

Answer:

10.7...........................................

A circular pool has a radius of 20 feet. What is the area of the pool? Use 3.14 for π.

Answers

Answer:

197.192

Step-by-step explanation:

The formula forthe area of a circle is 3.14r squared

so 3.14(20) 2(squared)=197.192

Can someone please help me only if you know how to do geometry thank you

Answers

Answer: SSS

The markings on the two triangles display side congruencey.

So AC is congruent to EC and etc. There are three pairs of side markings, so that means the two triangles are congruent because of the side, side side postulate.

F(x) = x^2-3x+18f(x)=x 2 −3x+18f, left parenthesis, x, right parenthesis, equals, x, squared, minus, 3, x, plus, 18 What is the value of the discriminant of fff? How many distinct real number zeros does fff have?

Answers

Answer:

No real roots or no real zeros.

Step-by-step explanation:

The given function is

[tex]f(x)=x^2-3x+18[/tex]

We need to find the number of real zeroes.

The given function is a quadratic function.

Let a quadratic function is [tex]f(x)=ax^2+bx+c[/tex].

If [tex]b^2-4ac<0[/tex], then the function has no real root.

If [tex]b^2-4ac=0[/tex], then the function has one real root with multiplicity 2.

If [tex]b^2-4ac>0[/tex], then the function has two real roots.

In the given function,

[tex]a=1,b=-3,c=18[/tex]

So,

[tex]b^2-4ac=(-3)^2-4(1)(18)=9-72=-63<0[/tex]

The value of discriminant is -63.

Therefore, the given function has no real root or no real zeros.

Answer:

The discriminant value of F is -63

F has 0 distinct real number zeros

Step-by-step explanation:

BIg Points big answer and explanations

Answers

Answer:

1/10

Step-by-step explanation:

We are hoping for 2 of the 20, and 2/20 simplified equals 1/10

C: 1/5 chance.

There are 20 possible outcomes, with 4 involving chicken and 4 involving ribs.

However, only 2 of each of those involves red sauce.  

So the possibilities of getting a red sauce with chicken or ribs is 4.

4/20 = 1/5.

Match each function with its domain. 1. S(x) = √x 2. H(x) = √2+x 3. Z(x) = √x-2 4. Q(x) = √2-x 5. V(x) = √-x 6. N(x) = ^3√2-x x ≤ 2 x ≥ -2 x ≥ 0 x ≥ 2 x ≤ 0 All real numbers

Answers

Answer:

1. x ≥ 0

2. x ≥ -2

3. x ≥ 2

4. x ≤ 2

5. x ≤ 0

6. All real numbers

Step-by-step explanation:

Domain of square function:

Suppose we have a square function in the following format:

[tex]f(x) = \sqrt[n]{g(x)}[/tex]

If n is even, the domain of f(x) is:

[tex]g(x) \geq 0[/tex]

Otherwise, if n is odd, the domain of f(x) is all real numbers.

1. S(x) = √x

If n does not appear is that it is 2.

g(x) = x

So the domain is:

[tex]x \geq 0[/tex]

2. H(x) = √2+x

g(x) = 2 + x

So

[tex]2 + x \geq 0[/tex]

[tex]x \geq -2[/tex]

3. Z(x) = √x-2

g(x) = x - 2

[tex]x - 2 \geq 0[/tex]

[tex]x \geq 2[/tex]

4. Q(x) = √2-x

g(x) = 2 - x

[tex]2 - x \geq 0[/tex]

[tex]-x \geq -2[/tex]

Multiplying by -1, everything

[tex]x \leq 2[/tex]

5. V(x) = √-x

g(x) = -x

Then

[tex]-x \geq 0[/tex]

Multiplying by -1

[tex]x \leq 0[/tex]

6. N(x) = ^3√2-x

Cubic root(odd number), so all real numbers.

Two Parallel lines are cut by a transversal and form a pair of alternate exterior angles. One angle measures (6x+5) and other measures (7x+4). Explain how to determine what those other angles actually measure.

Answers

Answer:

The other angles will measure 83.9° and 96.1°

Step-by-step explanation:

If two parallel lines are cut by a transversal and form a pair of alternate exterior angles, then the sum of the two exterior angles will be equal to 180° as shown;

If one angle measures (6x+5) and other measures (7x+4), then;

(6x+5)+(7x+4) = 180° (sum of two exterior angles)

Firstly, lets get the value of x from the equation.

6x+7x+9 = 180°

13x= 180-9

13x = 171

x = 171/13

x = 13.15°

The first angle 6x+5 will measure 6(13.15)+5 = 83.9°

The other angle 7x+4 will mesure 7(13.15)+4 = 96.1°

Answer:

Since the lines are parallel, the alternate exterior angles are congruent. Therefore you can set the expressions equal to each other and solve for x. Then you can substitute the value of x back into either expression to find the angle measure.

Step-by-step explanation: exact answer


[tex]x - 15 = - 6[/tex]

Answers

Answer:

The value of [tex]x[/tex] :

[tex]x = 9[/tex]

Step-by-step explanation:

-Solve for [tex]x[/tex] :

[tex]x =- 15 = -6[/tex]

-Add both sides by 15:

[tex]x - 15 + 15 = -6 + 15[/tex]

[tex]x = 9[/tex]

Now, you have found the answer.

Please help it’s geometry

Answers

The answer is SSS that’s probably wrong
The answer is SSS because they are all sides. SAS is side angle side, which you don’t have an angle.

Please answer correctly !!!!!!!! Will mark brainliest !!!!!!!!!

Answers

Answer:

C. Add the equations

Step-by-step explanation:

Adding both the equations will help eliminating 5y.

The angle \theta_1θ 1 ​ theta, start subscript, 1, end subscript is located in Quadrant \text{II}IIstart text, I, I, end text, and \sin(\theta_1)=\dfrac{1}{4}sin(θ 1 ​ )= 4 1 ​ sine, (, theta, start subscript, 1, end subscript, ), equals, start fraction, 1, divided by, 4, end fraction . What is the value of \cos(\theta_1)cos(θ 1 ​ )cosine, (, theta, start subscript, 1, end subscript, )?

Answers

Answer:

[tex]\cos(\theta_1)=-\dfrac{\sqrt{15}}{4}[/tex]

Step-by-step explanation:

The angle [tex]\theta_1[/tex] is located in Quadrant II.

[tex]\sin(\theta_1)=\dfrac{1}{4}[/tex]

From trigonometry, we know that:

[tex]\sin(\theta)=\dfrac{Opposite}{Hypotenuse}\\$Therefore:\\Opposite=1\\Hypotenuse=4\\Using Pythagorean theorem:\\Hypotenuse^2=Opposite^2+Adjacent^2\\4^2=1^2+Adjacent^2\\Adjacent^2=16-1\\Adjacent^2=15\\Adjacent=\sqrt{15}[/tex]

Now, in Quadrant II,

The x-axis is negativeThe y-axis is positive

Therefore, the Adjacent angle to [tex]\theta_1 =-\sqrt{15}[/tex]

Therefore:

[tex]\cos(\theta_1)=\dfrac{Adjacent}{Hypotenuse}=\dfrac{-\sqrt{15}}{4}\\\\\cos(\theta_1)=-\dfrac{\sqrt{15}}{4}[/tex]

The box plots represent the distributions of typing speeds of students before and after a computer-programming course.



Which statement is true about the variability of the distributions?

The interquartile range of the typing speeds after the course is greater than the interquartile range of the speeds before the course.
The interquartile ranges of the two distributions are the same.
The range of the speeds after the course is smaller than the range of the speeds before the course.
The ranges of the two distributions are the same.

Answers

Answer:

C on EDG

Step-by-step explanation:

Answer: C on edge

Step-by-step explanation:

the range of the speeds after the course is smaller than the range of the speeds before the course.

13 points! Easy question for y’all mathy people! Ez points! Question in photo.

Answers

Answer: 1, 2, and 4

Step-by-step explanation:

The graph with straight line is an function.

The second one shows about every has two single numbers instead of more than one output.

The third one shows the points, but -1 has more than one output, show it's not an function.

The final one shows the slope intercept form and it's just an function.

Answer:

1,2,4 also

Step-by-step explanation:

Richard drives his car for 5 hours.
His average speed is 30 mph.
(a) How far does Richard travel?
miles
Richard drives 560 km the next day.
(b) Work out how much further Richard travelled, compared to the previous day.
5 miles = 8 kilometers
miles

Answers

Answer:

A. 150 miles

B. 200 miles further

Step-by-step explanation:

Multiply 30 by 5 to find how far he traveled the first day

30 x 5 = 150 miles

For the second day, convert km to miles by making a proportion

[tex]\frac{5}{8}[/tex] = [tex]\frac{x}{560}[/tex]

x = 350 miles

Then, to find the difference, subtract the 350 - 150 = 200

What is the value of y when x= 2.5

Answers

Answer:

The value of y is unknown bas there is no equation

simplify 15a^6bc^4/35a^2c^4

Answers

Answer:

[tex]\dfrac{3a^4b}{7}[/tex]

Step-by-step explanation:

[tex]\dfrac{15a^6bc^4}{35a^2c^4}= \\\\\dfrac{15}{35}\times\dfrac{a^6}{a^2}\times\dfrac{c^4}{c^4}\times b= \\\\\dfrac{3}{7}\times a^4 \times 1 \times b=\\\\\boxed{\dfrac{3a^4b}{7}}[/tex]

Hope this helps!

Answer:

B

Step-by-step explanation:

You have a bag of 130 strawberries. You eat 27 strawberries and then split the remaining strawberries between 4 friends. Which of the following is a good estimation of how many strawberries each of your friends get?

Answers

Answer:

Step-by-step explanation: (130-27)/4 = 25.75 im not sure how you want to round it

Answer:25

Step-by-step explanation: to round 25.75 you have to round it 25 but i believe 25.75 is less than 5 we simply remove the fractional part to get the answer.25.Hope this helped.

Can someone answer this i need help

Answers

All that is needed to do is to simplify the expression.

The answer to your question is:

[tex]6 x^{6}[/tex] - [tex]10 x^{3}[/tex] + [tex]8x^{2}[/tex]

Hope I helped!

The answer is 6x^6 - 10x^3 + 8x^2

Betty is now three times as old as Mickey, but in five years she will be only twice as old as Mickey. What are the respective ages of Mickey and Betty now?

Answers

Answer:15

Step-by-step explanation:

it could be 15 but (three times) is geting me off

sorry if i was no help and all the number's wow

Janice already has $65 dollars in her savings account. If she puts $5 per week in her account, write an equation to find out how many weeks she must save to have at least $200 in her account.
2 points
Your answer

Answers

Answer: 65 + 5x ≥200

Step-by-step explanation:

Hi, to answer this question we have to write an inequality with the information given:

Janice's savings (65) plus the product of the amount she saves per week (5) and the number of weeks (x) must be higher or equal to $200.

Mathematically speaking:

65 + 5x ≥200

If we solve it for x:

5x ≥200-65

5x ≥135

x ≥ 135/5

x ≥ 27 weeks

Feel free to ask for more if needed or if you did not understand something.

Gracie is making 2 and 1/2 batches of chocolate chip cookies for her school bake sale. If one batch of cookies requires 1 and 1/4 cups of flour, how many cups of flour will she need?

Answers

Answer:

Step-by-step explanation:

What you just have to do is multiply 2 1/2 and 1 1/4, but i can still do it. 2 1/2 is the same as 5/2 , and 1 1/4 is the same as 5/4, so just multiply the top and the bottom and you will get 25/8 whicxh is basically 3 and 1/8

How can I solve this problem?

Answers

a) 108°

Answer:

b) 54°

Step-by-step explanation:

a) By arc sum property of a circle.

[tex] m\widehat {AEB} + m\widehat {AB} = 360°\\

252° + m\widehat {AB} = 360°\\

m\widehat {AB} = 360°- 252°\\

\huge \red {\boxed {m\widehat {AB} = 108°}} \\[/tex]

b) In the given figure, DC is tangent to the circle with O at point A and AB is chord or secant. Therefore, by tangent secant theorem:

[tex] m\angle BAC = \frac{1}{2} \times m\widehat {AB}\\\\

\therefore m\angle BAC = \frac{1}{2} \times 108°\\\\

\huge \purple {\boxed {m\angle BAC = 54°}} [/tex]

find the values for a and b.

Answers

Answer:

the value of a and b is 68

Step-by-step explanation:

the other half is 224 so you minus 360 from 224 which will give us 136 then you divide by 2 to get a and b.

Answer:

a = 68, b = 136

Step-by-step explanation:

The angle in the circle = 360°, thus

b = 360 - 224 = 136

The inscribed angle a between a tangent- chord is equal to half the measure of its intercepted arc, so

a = 0.5 × 136 = 68

Which algebraic expressions are polynomials

Answers

Answer:

2,3,5,6

Step-by-step explanation:

2,3,5,6 are the right answers

If 12 1/2% of a sum of money is $40, what is the total sum of money?

Answers

Answer:

i think its 320

Step-by-step explanation:

i looked it up

Answer:

320

Step-by-step explanation:

To find the total sum, divide the sum of money by the percentage.

12 1/2% = 0.125

40/0.125 = 320

Other Questions
Which of these is not affected by a person's credit score?A. Apartment rentalsB. Credit card ratesC. Ability to get a cell phone contractD. Federal income tax Read the following passage:Our democratic-republican government is based on theidea of the natural right of every individual member thereofto a voice and a vote in making and executing the laws.What rhetorical device does Anthony use in this passage?A. AnecdoteB. AllusionC. AntithesisD. Alliteration lan is working on a project report that will go through multiple rounds ofrevisions. He decides to establish a file labeling system to help keep track ofthe different versions. He labels the original documentProjectReport_v1.docx. How should he label the next version of thedocument?A. ProjectReport_revised.docxB. ProjectReport_v1_v2.docxC. Report_v2.docxD. ProjectReport_v2.docx what is this ^ called A projectile is fired with an initial speed of 37.7 m/s at an angle of 41.2 above the horizontal on a long flat firing range. Determine the maximum height reached by the projectile. Which is an example of a primary source?A. A letter written by a criminal while in jailB. A television report three months after an event happenedC. A book written about an eventD. An interview with an expert who was not involved Katrina is solving the equation x minus 2 (x + 3) = 4 (2 x + 3) minus (x + 4). Which equivalent equations might Katrina use? Check all that apply. x minus 2 x minus 6 = 8 x + 12 minus x minus 4 x minus 2 x + 6 = 8 x + 12 minus x + 4 x minus 2 x minus 6 = 8 x + 12 minus x + 4 x minus 2 x + 3 = 8 x + 3 minus x + 4 Negative x minus 6 = 7 x + 8 Negative x + 6 = 7 x + 16 Negative x minus 6 = 7 x + 16 Negative x + 3 = 7 x + 7 Angie bought 44 pencils and paid 66.00. She sold the pencils at a rate of 2 for $3.50calculate her profit. HURRY PLEASE! :)Employers prefer to hire creative, resourceful people. At times, in order to accomplish their goals, some employees may perform actions that some people may consider as being creatively resourceful and others may consider unethical. Discuss creative resourcefulness and ethics, with examples where you think people have been creatively resourceful and ethical and examples where you think the person has crossed the line between ethics and resourcefulness. You earn $15 every week .at this rate , how much will you earn in 5 weeks mention the objectives of physical education? A group of fitness club members lose a combined total of 28 kilograms in 1 week. There are approximately 2.2 pounds in 1 kilogram l. Assuming the weight loss happened at a constant rate, about how many pounds did the group lose each day? HELP!!! WILL MARK BRAINLIEST!!!The following graph describes function 1, and the equation below it describes function 2. Determine which function hasa greater maximum value, and provide the ordered pair.Function 1Function 2f(x) = -x2 + 2x - 3Function 1 has the larger maximum at (1,4).Function 1 has the larger maximum at (4,1).Function 2 has the larger maximum at (1, -2).Function 2 has the larger maximum at (-2, 1). y = -x2 + 6x - 5 whats the axis of symmetry. Use the method of combining like terms to find the equivalent expression to 6u+2v+3u7v2. Ben attended a workshop hosted by SCORE, a SBA organization that counsels small-business owners on developing a good vision statement. In that workshop the speakers discussed how an organization's vision needs to describe ________. Suppose you deposit $1000 in an account paying 3% annual interest, compounded continuously. Use A=Pe^rt to find the balance after 10 years Alex is making a milkshake.She needs 1/3 cups of milk.She decided to add 1/4 more cups.How much milk is she putting into her milk shake?Be sure to show your equation One angle of a triangle is obtuse. The sum of the measures of the other angles must be A.more than 90 degrees B.less than 90 degrees C.equal to 90 degrees help i need this 10 points please \frac{(2+4)^{4} }{2}Plz Answer